Weaken Questions - - Question 63
A recent report on an environmental improvement program was criticized for focusing solely on pragmatic solutions to ...
Replies
Annie March 10, 2020
Hi @odsimkins,This question asks you to find the answer choice which "best" serves the critics argument. This means that multiple answer choices may serve the argument, but you need to pick the one which is most helpful to them.
(A) is incorrect because it only tangentially supports the argument. Say that the government provides 95% of the funding, the program will still need to have a reputation for competence to get that and that is necessary for it to run.
(B) is correct because it directly counters the argument that the report can only focus on the problems rather than a coherent vision. This statement tells us that a coherent vision is necessary to eliminating the problems, so the report was definitely wrong not to include one.
(C) is incorrect because it is irrelevant. We are talking about the need for a coherent vision now, we do not care what the state of its vision has been in the past.
(D) is incorrect because it is still possible that the government will do so. This does not help with the argument that the report shouldn't have focused on solving the problems.
(E) is incorrect because it's reputation is what is necessary for it's funding. It is irrelevant if that reputation is not fair.
shunhe March 10, 2020
Hi @odsimkins,Thanks for the question! (A) is wrong because it doesn’t help out the critics of the report. Whether or not the government provides 100%, it might still be the case that government funding is needed for the program to continue. The government could, for example, provide 99%, and then government funding would actually still be necessary.
(C) is wrong because if the program doesn’t have a coherent vision anymore, then the authors of the report’s position is actually boosted, since the program likely wouldn’t have a reputation for competence.
(D) is wrong because it doesn’t talk about why the government has cut off funding for the program, and so doesn’t weaken the report’s authors position.
(E) is wrong because if the program has a worse reputation for incompetence than it deserves, then it’s probably true that the program does need to regain a reputation for competence, which is what the report’s authors are saying. (E) definitely doesn’t undermine their position.
Hope this helps! Feel free to ask any other questions that you might have.